Add last few weeks of Phys 102 solutions.
[course.git] / latex / problems / Serway_and_Jewett_8 / problem29.37.tex
1 \begin{problem*}{29.37}
2 A rod of mass $0.720\U{kg}$ and radius $6.00\U{cm}$ rests on two
3 parallel rails (Fig.~P29.37) that are $d=12.0\U{cm}$ apart and
4 $L=45.0\U{cm}$ long.  The rod carries a current of $I=48.0\U{A}$ in
5 the direction shown and rolls along the rails without slipping.  A
6 uniform magnetic field of magnitude $0.240\U{T}$ is directed
7 perpendicular to the rod and rails.  If it starts from rest, what is
8 the speed of the rod as it leaves the rails?
9 \end{problem*}
10
11 \begin{solution}
12 \end{solution}